Γ΄ Παγκύπριος Διαγωνισμός Επιλογής για JBMO, 2017

Συντονιστές: cretanman, ΔΗΜΗΤΡΗΣ ΙΩΑΝΝΟΥ, socrates

Άβαταρ μέλους
Soteris
Δημοσιεύσεις: 466
Εγγραφή: Δευ Ιούλ 21, 2014 1:59 pm
Τοποθεσία: Λάρνακα, Κύπρος

Γ΄ Παγκύπριος Διαγωνισμός Επιλογής για JBMO, 2017

#1

Μη αναγνωσμένη δημοσίευση από Soteris » Σάβ Απρ 29, 2017 2:04 pm

Πρόβλημα 1

Να βρείτε τον μεγαλύτερο θετικό ακέραιο \displaystyle{\nu}, ο οποίος διαιρεί τον \displaystyle{p^6-1} για όλους τους πρώτους \displaystyle{p>7}.

Πρόβλημα 2

Αν \displaystyle{a, \beta, \gamma>0} πραγματικοί αριθμοί με \displaystyle{a+\beta+\gamma=1}, να δείξετε ότι:

(α) \displaystyle{\sqrt{a+\beta\gamma}+\sqrt{\beta+a\gamma}+\sqrt{\gamma+a\beta}\leqslant 2}

(β) \displaystyle{\dfrac{a^2}{\sqrt{a+\beta\gamma}}+\dfrac{\beta^2}{\sqrt{\beta+a\gamma}}+\dfrac{\gamma^2}{\sqrt{\gamma+a\beta}}\geqslant\dfrac{1}{2}}

Πρόβλημα 3

Δίνεται οξυγώνιο τρίγωνο \displaystyle{\vartriangle{\rm AB\Gamma}} με \displaystyle{{\rm AB}<{\rm A\Gamma}}. Οι κάθετες από τις κορυφές \displaystyle{\rm B} και \displaystyle{\rm \Gamma} προς τις απέναντι πλευρές \displaystyle{{\rm A\Gamma} και \displaystyle{{\rm AB}}, αντίστοιχα, τέμνονται στο σημείο \displaystyle{{\rm H}}. Οι μεσοκάθετες των πλευρών του τριγώνου τέμνονται στο σημείο \displaystyle{{\rm O}}. Ονομάζουμε \displaystyle{{\rm \Delta}} το μέσο της πλευράς \displaystyle{{\rm B\Gamma}}. Προεκτείνουμε την \displaystyle{{\rm O\Delta}} προς το \displaystyle{{\rm \Delta}} και στην προέκτασή της παίρνουμε σημείο \displaystyle{{\rm O’}} τέτοιο ώστε \displaystyle{{\rm O\Delta}={\rm \Delta O’}}. Υποθέτουμε ότι οι ευθείες \displaystyle{{\rm HO’}} και \displaystyle{{\rm A\Delta}} τέμνονται στο σημείο \displaystyle{{\rm P}}. Από το \displaystyle{{\rm H}} φέρουμε κάθετη προς την \displaystyle{{\rm A\Delta}} και έστω \displaystyle{{\rm K}} το ίχνος της πάνω στην \displaystyle{{\rm A\Delta}}. Η ευθεία \displaystyle{{\rm \Gamma K}} τέμνει την \displaystyle{{\rm AB}} στο σημείο \displaystyle{{\rm T}}. Να αποδείξετε ότι οι γωνίες \displaystyle{\angle{\rm BT\Gamma}} και \displaystyle{\angle{\rm PHK}} είναι ίσες.

Πρόβλημα 4

Ο Γιώργος και ο Δημήτρης παίζουν το εξής παιχνίδι:

Αρχικά είναι γραμμένος στον πίνακα ο αριθμός \displaystyle{2017}. Ξεκινώντας από τον Γιώργο και παίζοντας εναλλάξ, κάθε παίκτης σβήνει τον αριθμό που βλέπει στον πίνακα και στην θέση του γράφει έναν μη αρνητικό ακέραιο, ο οποίος προκύπτει από τον προηγούμενο αριθμό αφαιρώντας ένα τέλειο τετράγωνο θετικού ακεραίου, το οποίο δεν είναι πολλαπλάσιο του \displaystyle{5}. Ο παίκτης που θα γράψει πρώτος στον πίνακα τον αριθμό \displaystyle{0} κερδίζει το παιχνίδι. Να βρείτε ποιος από τους δύο παίκτες έχει στρατηγική νίκης.


Σωτήρης Λοϊζιάς

Λέξεις Κλειδιά:
Γιάννης Μπόρμπας
Δημοσιεύσεις: 217
Εγγραφή: Τρί Δεκ 13, 2016 10:41 pm
Τοποθεσία: Χανιά

Re: Γ΄ Παγκύπριος Διαγωνισμός Επιλογής για JBMO, 2017

#2

Μη αναγνωσμένη δημοσίευση από Γιάννης Μπόρμπας » Σάβ Απρ 29, 2017 2:27 pm

Soteris έγραψε:Πρόβλημα 1

Να βρείτε τον μεγαλύτερο θετικό ακέραιο \displaystyle{\nu}, ο οποίος διαιρεί τον \displaystyle{p^6-1} για όλους τους πρώτους \displaystyle{p>7}.
Παίρνοντας p=11 και p=13 έχουμε:
11^6-1=2^3\times 3^2\times 5\times 7\times 19\times 37
13^6-1=2^3\times 3^2\times 7\times 61\times 157
Ο μέγιστος κοινός διαιρέτης των παραπάνω αριθμών είναι ο: 2^3\times 3^2\times 7
Θα δείξουμε ότι ο μεγαλύτερος αριθμός είναι ο 504
Από FLT: p^6-1\equiv 1-1=0(\mod 7)
Στην συνέχεια αν p\equiv 1(\mod 3) τότε 3|p-1 και 3|p^2+p+1 άρα 9|p^6-1 αν p\equiv 2(\mod 3) τότε 3|p+1 και 3|p^2-p+1 άρα 9|p^6-1
Τέλος (p-1)(p+1)=p^2-1|p^6-1
και αν p\equiv 1(\mod 4) τότε 4|p-1 και 2|p+1 άρα 8|p^6-1
αν p\equiv 3(\mod 4) τότε 4|p+1 και 2|p-1 άρα 8|p^6-1
Οπότε ο ζητούμενος αριθμός είναι ο 2^3\times 3^2\times 7=504=n
τελευταία επεξεργασία από Γιάννης Μπόρμπας σε Σάβ Απρ 29, 2017 2:48 pm, έχει επεξεργασθεί 3 φορές συνολικά.


Γιάννης Μπορμπαντωνάκης
Άβαταρ μέλους
Ορέστης Λιγνός
Δημοσιεύσεις: 1835
Εγγραφή: Κυρ Μάιος 08, 2016 7:19 pm
Τοποθεσία: Χαλάνδρι Αττικής
Επικοινωνία:

Re: Γ΄ Παγκύπριος Διαγωνισμός Επιλογής για JBMO, 2017

#3

Μη αναγνωσμένη δημοσίευση από Ορέστης Λιγνός » Σάβ Απρ 29, 2017 2:28 pm

Soteris έγραψε: Πρόβλημα 2

Αν \displaystyle{a, \beta, \gamma>0} πραγματικοί αριθμοί με \displaystyle{a+\beta+\gamma=1}, να δείξετε ότι:

(α) \displaystyle{\sqrt{a+\beta\gamma}+\sqrt{\beta+a\gamma}+\sqrt{\gamma+a\beta}\leqslant 2}

(β) \displaystyle{\dfrac{a^2}{\sqrt{a+\beta\gamma}}+\dfrac{\beta^2}{\sqrt{\beta+a\gamma}}+\dfrac{\gamma^2}{\sqrt{\gamma+a\beta}}\geqslant\dfrac{1}{2}}
Για ευκολία, αντί a, \beta, \gamma, γράφω a,b,c.

α) Είναι

\displaystyle \sum{\sqrt{a+bc}}=\sum \sqrt{1-b-c+bc}=\sum \sqrt{(1-b)(1-c)} \leqslant \sum \dfrac{(1-b)+(1-c)}{2}=

\dfrac{2-b-c}{2}+\dfrac{2-a-c}{2}+\dfrac{2-a-b}{2} \mathop = \limits^{a+b+c=1}2.

β) Από Cauchy-Schwarz, \displaystyle \sum \dfrac{a^2}{\sqrt{a+bc}} \geqslant \dfrac{(a+b+c)^2}{\sum \sqrt{a+bc}}  \mathop \geqslant \dfrac{4}{2}=2.


Κερδίζουμε ό,τι τολμούμε!
harrisp
Δημοσιεύσεις: 546
Εγγραφή: Σάβ Μαρ 28, 2015 8:49 pm

Re: Γ΄ Παγκύπριος Διαγωνισμός Επιλογής για JBMO, 2017

#4

Μη αναγνωσμένη δημοσίευση από harrisp » Σάβ Απρ 29, 2017 2:35 pm

2)

(α) Ισχυει a+bc\leq \dfrac {4a+(1-a)^2}{4}= \dfrac {(a+1)^2}{4}

Αρα LHS\leq \dfrac {a+b+c+3}{2}=2 οεδ. Ισότητα για a=b=c=1/3

(β) Απο Andreescu επεται άμεσα το ζητούμενο!



Edit: Με πρόλαβε ο Ορέστης! Γεια σου Ορέστη!


Άβαταρ μέλους
Ορέστης Λιγνός
Δημοσιεύσεις: 1835
Εγγραφή: Κυρ Μάιος 08, 2016 7:19 pm
Τοποθεσία: Χαλάνδρι Αττικής
Επικοινωνία:

Re: Γ΄ Παγκύπριος Διαγωνισμός Επιλογής για JBMO, 2017

#5

Μη αναγνωσμένη δημοσίευση από Ορέστης Λιγνός » Σάβ Απρ 29, 2017 5:07 pm

Soteris έγραψε: Πρόβλημα 3

Δίνεται οξυγώνιο τρίγωνο \displaystyle{\vartriangle{\rm AB\Gamma}} με \displaystyle{{\rm AB}<{\rm A\Gamma}}. Οι κάθετες από τις κορυφές \displaystyle{\rm B} και \displaystyle{\rm \Gamma} προς τις απέναντι πλευρές \displaystyle{{\rm A\Gamma} και \displaystyle{{\rm AB}}, αντίστοιχα, τέμνονται στο σημείο \displaystyle{{\rm H}}. Οι μεσοκάθετες των πλευρών του τριγώνου τέμνονται στο σημείο \displaystyle{{\rm O}}. Ονομάζουμε \displaystyle{{\rm \Delta}} το μέσο της πλευράς \displaystyle{{\rm B\Gamma}}. Προεκτείνουμε την \displaystyle{{\rm O\Delta}} προς το \displaystyle{{\rm \Delta}} και στην προέκτασή της παίρνουμε σημείο \displaystyle{{\rm O’}} τέτοιο ώστε \displaystyle{{\rm O\Delta}={\rm \Delta O’}}. Υποθέτουμε ότι οι ευθείες \displaystyle{{\rm HO’}} και \displaystyle{{\rm A\Delta}} τέμνονται στο σημείο \displaystyle{{\rm P}}. Από το \displaystyle{{\rm H}} φέρουμε κάθετη προς την \displaystyle{{\rm A\Delta}} και έστω \displaystyle{{\rm K}} το ίχνος της πάνω στην \displaystyle{{\rm A\Delta}}. Η ευθεία \displaystyle{{\rm \Gamma K}} τέμνει την \displaystyle{{\rm AB}} στο σημείο \displaystyle{{\rm T}}. Να αποδείξετε ότι οι γωνίες \displaystyle{\angle{\rm BT\Gamma}} και \displaystyle{\angle{\rm PHK}} είναι ίσες.
Για ευκολία, χρησιμοποιώ αγγλικά γράμματα.

Προφανώς, στο τετράπλευρο BOCO', οι διαγώνιες διχοτομούνται, άρα είναι παραλληλόγραμμο. Έτσι, \widehat{BO'C}=\widehat{BOC}=2\widehat{A}.

Ακόμη, είναι προφανές ότι \widehat{BHC}=180^0-\widehat{A}.

[Παίρνουμε σημείο X στο επίπεδο (όπως στο σχήμα), ώστε το BHCX να είναι εγγράψιμο.

Τότε, \widehat{BXC}=180^0-\widehat{BHC}=\widehat{A}=\dfrac{\widehat{BO'C}}{2}.

Άρα, στο τρίγωνο BXC, το σημείο O' είναι τέτοιο ώστε BO'=O'C, \, \widehat{BO'C}=2\widehat{BXC}, οπότε είναι το περίκεντρο του.

Το BHCX είναι εγγράψιμο, άρα το O' είναι το κέντρο του περιγεγραμμένου κύκλου του, οπότε O'B=O'H=O'C.]


Από γνωστή ιδιότητα, AH=2OD=2OD'.

Στο τρίγωνο AHP, είναι AH \parallel = 2DO', οπότε AD=DP, HO'=O'P.

Από την AD=DP, έπεται ότι στο τετράπλευρο ABPC οι διαγώνιες διχοτομούνται, άρα αυτό είναι παραλληλόγραμμο, οπότε BT \equiv BA \parallel CP \Leftrightarrow \boxed{\widehat{BTC}=180^0-\widehat{KCP}} (1).

Από BT \parallel CP, CH \perp BT \Rightarrow CH \perp CP, και αφού HK \perp KP, το HKCP είναι εγγράψιμο.

Άρα, \boxed{\widehat{KHP}=180^0-\widehat{KCP}} (2).

Από (1), (2), το ζητούμενο είναι άμεσο.
JBMO-KYPROY.png
JBMO-KYPROY.png (49.17 KiB) Προβλήθηκε 1889 φορές
Υ.Γ. Τώρα βλέπω ότι οι λέξεις στις κόκκινες αγκύλες είναι περιττές, χωρίς να επηρεάζουν την υπόλοιπη λύση. Τις αφήνω για τα παραπάνω στοιχεία που αποδείχθηκαν.


Κερδίζουμε ό,τι τολμούμε!
Άβαταρ μέλους
Ορέστης Λιγνός
Δημοσιεύσεις: 1835
Εγγραφή: Κυρ Μάιος 08, 2016 7:19 pm
Τοποθεσία: Χαλάνδρι Αττικής
Επικοινωνία:

Re: Γ΄ Παγκύπριος Διαγωνισμός Επιλογής για JBMO, 2017

#6

Μη αναγνωσμένη δημοσίευση από Ορέστης Λιγνός » Κυρ Απρ 30, 2017 1:04 am

Soteris έγραψε:
Πρόβλημα 4

Ο Γιώργος και ο Δημήτρης παίζουν το εξής παιχνίδι:

Αρχικά είναι γραμμένος στον πίνακα ο αριθμός \displaystyle{2017}. Ξεκινώντας από τον Γιώργο και παίζοντας εναλλάξ, κάθε παίκτης σβήνει τον αριθμό που βλέπει στον πίνακα και στην θέση του γράφει έναν μη αρνητικό ακέραιο, ο οποίος προκύπτει από τον προηγούμενο αριθμό αφαιρώντας ένα τέλειο τετράγωνο θετικού ακεραίου, το οποίο δεν είναι πολλαπλάσιο του \displaystyle{5}. Ο παίκτης που θα γράψει πρώτος στον πίνακα τον αριθμό \displaystyle{0} κερδίζει το παιχνίδι. Να βρείτε ποιος από τους δύο παίκτες έχει στρατηγική νίκης.
Στην αρχή, στον πίνακα είναι γραμμένος ο αριθμός 2017.

Ο Γιώργος, αφαιρεί ένα τέλειο τετράγωνο από τον 2017, και γράφει στον πίνακα τον αριθμό G_1.
Ο Δημήτρης, αφαιρεί ένα τέλειο τετράγωνο από τον G_1, και γράφει στον πίνακα τον αριθμό D_1.

Εδώ τελειώνει ο πρώτος γύρος.

Ακολουθούν και οι άλλοι γύροι, όπως φαίνεται στο σχήμα, μέχρι να τελειώσει το παιχνίδι.
kyprosJBMO4.png
kyprosJBMO4.png (7.44 KiB) Προβλήθηκε 1795 φορές
Θα δείξουμε ότι ο Δημήτρης έχει στρατηγική νίκης όταν D_1 \equiv D_2 \equiv \ldots \equiv D_n  (\bmod5).

Θα δούμε τώρα πώς μπορεί να επιτευχθεί αυτό: έστω ότι ο Γιώργος στην αρχή του κάθε γύρου παίρνει από τον πίνακα τον αριθμό N \in (2017,D_1, D_2, \ldots D_n) και έστω ότι αφαιρεί το x^2. Τότε ο Δημήτρης παίρνει τον αριθμό N-x^2, και έστω ότι αφαιρεί το y^2.

Άρα, ο αριθμός που μένει στον πίνακα στο τέλος του κάθε γύρου είναι ο N-x^2-y^2, και πρέπει N \equiv N-x^2-y^2 (\bmod 5), οπότε x^2+y^2 \equiv 0(\bmod5).

Αφού όμως x,y \neq 0 (\bmod5) (από εκφώνηση), πρέπει x^2 \equiv 1(\bmod 5), y^2 \equiv 4(\bmod5) ή το αντίστροφο.

Άρα, όταν ο Γιώργος επιλέγει x ώστε x^2 \equiv 1 (\bmod 5), ο Δημήτρης επιλέγει y ώστε y^2 \equiv 4(\bmod5) και αντίστροφα.

Όμως, ο αριθμός που αρχίζει η όλη διαδικασία αρχίζει από τον αριθμό 2017, άρα πρέπει D_1 \equiv D_2 \equiv \ldots \equiv D_n \equiv N \equiv 2(\bmod5).

Άρα, η τελευταία κόκκινη κουκίδα, δηλαδή ο αριθμός που αφήνει στον πίνακα ο Δημήτρης, θα είναι ο 2.

Έτσι ο Γιώργος, παίρνει το 2, αφαιρεί (υποχρεωτικά), το 1 και γράφει στον πίνακα το 1.

Τελικά, ο Δημήτρης, αφαιρεί 1 και κερδίζει.


Κερδίζουμε ό,τι τολμούμε!
Άβαταρ μέλους
Ορέστης Λιγνός
Δημοσιεύσεις: 1835
Εγγραφή: Κυρ Μάιος 08, 2016 7:19 pm
Τοποθεσία: Χαλάνδρι Αττικής
Επικοινωνία:

Re: Γ΄ Παγκύπριος Διαγωνισμός Επιλογής για JBMO, 2017

#7

Μη αναγνωσμένη δημοσίευση από Ορέστης Λιγνός » Κυρ Απρ 30, 2017 11:21 am

Soteris έγραψε:
Αν \displaystyle{a, \beta, \gamma>0} πραγματικοί αριθμοί με \displaystyle{a+\beta+\gamma=1}, να δείξετε ότι:

(α) \displaystyle{\sqrt{a+\beta\gamma}+\sqrt{\beta+a\gamma}+\sqrt{\gamma+a\beta}\leqslant 2}
Κάπως αλλιώς:

Είναι 1=(a+b+c)^2 \geqslant 3(ab+bc+ca), άρα ab+bc+ca \leqslant \dfrac{1}{3}.

Από Cauchy Schwarz, \displaystyle \sum \sqrt{a+bc} \leqslant \sqrt{3(a+b+c+ab+bc+ca)} \leqslant \sqrt{3 \cdot \dfrac{4}{3}}=2 ό.έ.δ, με το = για a=b=c=\dfrac{1}{3}.

Υ.Γ. Είναι το Πρόβλημα 1 εδώ.


Κερδίζουμε ό,τι τολμούμε!
Xriiiiistos
Δημοσιεύσεις: 219
Εγγραφή: Τρί Μάιος 15, 2018 4:36 pm

Re: Γ΄ Παγκύπριος Διαγωνισμός Επιλογής για JBMO, 2017

#8

Μη αναγνωσμένη δημοσίευση από Xriiiiistos » Σάβ Ιαν 26, 2019 2:16 pm

Soteris έγραψε:
Σάβ Απρ 29, 2017 2:04 pm

Πρόβλημα 2

Αν \displaystyle{a, \beta, \gamma>0} πραγματικοί αριθμοί με \displaystyle{a+\beta+\gamma=1}, να δείξετε ότι:

(α) \displaystyle{\sqrt{a+\beta\gamma}+\sqrt{\beta+a\gamma}+\sqrt{\gamma+a\beta}\leqslant 2}



Έχει απαντηθεί αλλά αφήνω άλλη μία λύση για τον τρόπο σκέψης

Έστω a=\frac{k}{k+l+m},b=\frac{l}{k+l+m},c=\frac{m}{k+l+m}

Έχουμε \sqrt{a+bc}=\frac{\sqrt{k^{2}+kl+km+lm}}{k+l+m}=\frac{\sqrt{(k+l)(k+m)}}{k+l+m}\leq \frac{k+\frac{m+l}{2}}{k+l+m} Όμοια έχουμε \sqrt{b+ac}\leq \frac{l+\frac{m+c}{2}}{k+l+m},\sqrt{c+ab}\leq \frac{m+\frac{l+c}{2}}{k+l+m} και μετά προσθέτουμε και βγαίνει ο ζητούμενο, Η μόνη ανισότητα που χρησιμοποιήθηκε είναι η \sqrt{xy}\leq \frac{x+y}{2}


Απάντηση

Επιστροφή σε “Θέματα διαγωνισμών (ΕΜΕ, ΚΥΜΕ, BMO, JBMO, IMO, Kangaroo κλπ)”

Μέλη σε σύνδεση

Μέλη σε αυτήν τη Δ. Συζήτηση: Δεν υπάρχουν εγγεγραμμένα μέλη και 9 επισκέπτες